证明:若a≡b(mod m),那么a^n≡b^n(mod m),(其中n为非0自然数).

来源:学生作业帮助网 编辑:作业帮 时间:2024/04/27 14:03:26
证明:若a≡b(mod m),那么a^n≡b^n(mod m),(其中n为非0自然数).

证明:若a≡b(mod m),那么a^n≡b^n(mod m),(其中n为非0自然数).
证明:若a≡b(mod m),那么a^n≡b^n(mod m),(其中n为非0自然数).

证明:若a≡b(mod m),那么a^n≡b^n(mod m),(其中n为非0自然数).
a≡b(mod m),(a-b)|m,
a^n-b^n=(a-b)(a^n-1+(a^n-2)b+……+b^n-1)|m
所以a^n≡b^n(mod m)

证明:若a≡b(mod m),那么a^n≡b^n(mod m),(其中n为非0自然数). 举例证明同余的乘方性质:如果a ≡ b (mod m),那么a^n ≡ b^n (mod m) 求大神详细证明一个同余的式子 a≡b mod n那么a^2≡b^2 mod na≡b mod n那么a^2≡b^2 mod n求大神证明. 同余式a≡b(mod m)成立,a²≡b²(mod m)成立吗?如何证明?如题 如何证明性质7:若ac≡bc(mod m),(c,m)=1,那么a≡b(mod m),(记号(c,m)表示c与m的最大公约数性质7:若ac≡bc(mod m),(c,m)=1,那么a≡b(mod m),(记号(c,m)表示c与m的最大公约数),如何 同余乘方证明证明:(应用数学归纳法证明)(1)当n=1时,命题显然成立;(2)假设当n=k时,a^k≡b^k (mod m)成立,即a^k-b^k能被m整除.那么当n=k+1时∵a≡b (mod m)∴a=b+km (k是整数)∵a^(k+1)-b^(k+1)=a^(k+1) 如何证明 同余定理 中的 除法原理?除法原理:a ≡ b mod(cn) ==> a ≡ b mod(n); 求教如何证明? 同余的性质证明若ac ≡ bc (mod m) =0 则 a≡ b (mod m/(c,m)) 其中(c,m)表示c,m的最大公约数.请问同余的这个性质该怎么证明 数论题 证明:若n整除(a^n-b^n),则n整除(a^n-b^n)/(a-b),其中a,b,n均为整数.等价表述:若a^n-b^n≡0(mod n) ,则(a^n-b^n)/(a-b)≡0(mod n),其中a,b,n均为整数.(当n为素数时很容易证明,但这里要求n为整数,我就 (a*b)mod n与(a mod n)*(b mod n) 是否相等 设a≡b(mod m),c≡d(mod m),求证ac≡bd(mod m)设a≡b(mod m),c≡d(mod m)求证ac≡bd(mod m) 欧拉定理证明中:{既然这样,那么(a*x1 × a*x2×...×a*xφ(n))(mod n)= (a*x1(mod n) × a*x2(mod n) × ...× a*xφ(n)(mod n))(mod n)= (x1 × x2 × ...× xφ(n))(mod n)考虑上面等式左边和右边左边等于(a*(x1 × x2 基本同余定理证明【定义】设m是大于1的正整数,a,b是整数,如果m|(a-b),则称a与b关于模m同余,记作a≡b(mod m),读作a与b对模m同余.显然,有如下事实(1)若a≡0(mod m),则m|a;(2)a≡b(mod m)等价于a与b分别用m 整除,取余1.假设a和b都不被3和7整除,证明a^6=b^6(mod21)2.找出方程x^3=17(mod99)的所有解3.方程组x=a(mod m),y=b(mod n).证明如果gcd(m,n)|(a-b),那么这个方程组有一个唯一解整除mn/(gcd(m,n))4.求5x^2+x-7y+6=0的所 a,b,k为大于2的正整数a^k mod (k+1)=n;b^k mod (k+1)=m; 证明 n*m mod (k+1)=1;的充要条件为n=1,m=1;a,b,k为大于2的正整数a^k mod (k+1)=n;b^k mod (k+1)=m;证明 n*m mod (k+1)=1;的充要条件为n=1,m=1;充分条件不用证了,谁 a≡m(mod d) a^2 ≡n(mod d) 其中m,n什么关系?a≡m(mod d) a^2 ≡n(mod d)麻烦再给一些关于同余 、余数的定理 性质 (a+b) mod n 和[(a mod n) +b]mod n 有什么区别?(a+b) mod n 和[(a mod n) +b]mod n 有什么区别?结果一样么? a ≡ a (mod m) 若a ²≡ a (mod m) ,用同余式相乘,得到a三次方 ≡ a ² ≡ a (mod m)最后得到a的n次方 ≡ a (mod m) 行不?有啥条件限制的?数论中有这样的公式和类似的定义吗?